Tải bản đầy đủ (.pdf) (46 trang)

graph 2017 hai phong

Bạn đang xem bản rút gọn của tài liệu. Xem và tải ngay bản đầy đủ của tài liệu tại đây (817.39 KB, 46 trang )

ĐỒ THỊ
Lê Đức Thịnh – THPT Chuyên Trần Phú, Hải Phòng
A – MỞ ĐẦU:
Trong các kỳ thi Olympic Toán, Tổ hợp – Rời rạc là mảng được coi là khó nhất. Các
bài toán Tổ hợp – Rời rạc rất đa dạng, phong phú. Có nhiều bài toán Tổ hợp – Rời rạc đòi
hỏi khả năng tư duy thông minh, nhạy bén, sáng tạo, thậm chí là mang tính đột phá; lại có
những bài toán đòi hỏi cả độ dày kiến thức, cả khả năng sử dụng các định lý, tính chất có
tính công cụ để hỗ trợ.
Trong mảng Tổ hợp – Rời rạc, Đồ thị là phần mà đa số các em học sinh (và cả các
thầy cô giáo) còn rất e ngại. Phần này không chỉ bao gồm những bài toán ở dạng phát biểu
dưới ngôn ngữ Đồ thị cho dễ trình bày mà còn bao gồm cả những bài toán đòi hỏi vận dụng
tương đối nhiều và rộng các định nghĩa, tính chất, định lý, thậm chí là những kết quả “mạnh”
để giải quyết.
Trong bài viết chuyên đề này, với mong muốn giúp học sinh có được nền tảng kiến
thức, định hướng và khả năng vận dụng tốt hơn về phần Đồ thị, tác giả đã hệ thống các bài
tập theo các nhóm Lý thuyết và Phương pháp giải toán tương ứng. Các phần Lý thuyết ở
đây, tác giả không có tham vọng trình bày quá nhiều định nghĩa, kết quả bổ sung (điều này
độc giả có thể tìm hiểu chi tiết hơn ở các cuốn giáo trình khác) mà chỉ đưa ra các khái niệm,
kết quả phù hợp với các kỳ thi Olympic (tất nhiên là theo quan điểm cá nhân); chẳng hạn
như là việc chứng minh lại các bổ đề, định lý cần thiết phải hợp lý với thời lượng làm bài và
điều kiện phòng thi; hay là các kết quả đưa ra phải có tần suất xuất hiện tương đối trong các
đề thi. Các bài tập cũng được tác giả lựa chọn theo các mức độ khó tăng dần và xoay quanh
các đề thi Olympic các Quốc gia, Khu vực và Quốc tế trong các năm gần đây, cụ thể là từ
2013 đến 2017.
B – NỘI DUNG:
I – Mở đầu về đồ thị:
-) Đơn đồ thị vô hướng G V , E  bao gồm V là tập các đỉnh, E là tập các cặp có thứ tự gồm
hai phần tử khác nhau của V gọi là các cạnh.

Đơn đồ thị vô hướng
-) Đơn đồ thị có hướng G V , E  bao gồm V là tập các đỉnh, E là tập các cặp có thứ tự gồm


hai phần tử của V gọi là các cung.


Đơn đồ thị có hướng
-) Hai đỉnh u và v của đồ thị vô hướng G V , E  được gọi là kề nhau nếu  u , v  là cạnh
thuộc đồ thị G . Nếu e   u , v  là cạnh của đồ thị G thì ta nói cạnh này liên thuộc với hai
đỉnh u và v , hoặc ta nói cạnh e nối đỉnh u với đỉnh v , đồng thời các đỉnh u và v sẽ được
gọi là đỉnh đầu của cạnh  u , v  .
-) Bậc của đỉnh v trong đồ thị vô hướng, ký hiệu là deg v , là số cạnh liên thuộc với nó.
Đỉnh bậc 0 được gọi là đỉnh cô lập. Đỉnh bậc 1 được gọi là đỉnh treo.
-) Bổ đề bắt tay:
Giả sử G V , E  là đồ thị vô hướng với E cạnh. Khi đó 2 E   deg v .
vV

Chứng minh:
Rõ ràng mỗi cạnh e   u , v  bất kỳ, được tính một lần trong deg u và một lần trong deg v .
Từ đó suy ra số tổng tất cả các bậc bằng hai lần số cạnh.
-) Hệ quả:
Trong đồ thị vô hướng G V , E  số các đỉnh bậc lẻ là một số chẵn.
Chứng minh:
Gọi O là tập các đỉnh bậc chẵn và V là tập các đỉnh bậc lẻ. Từ bổ đề bắt tay ta suy ra:
2m   deg v   deg v   deg v
vV

vO

vU

Do deg v là chẵn với v là đỉnh trong O nên tổng thứ hai trong vế phải cũng là một số chẵn.
-) Nếu e   u , v  là cung của đồ thị có hướng G thì ta nói hai đỉnh u và v là kề nhau, và nói

cung  u , v  nối đỉnh u với đỉnh v hoặc cũng nói cung này đi ra khỏi đỉnh u và đi vào đỉnh

v . Đỉnh u ( v ) sẽ được gọi là đỉnh đầu (cuối) của cung  u , v  .
-) Ta gọi bậc ra (bậc vào) của đỉnh v trong đồ thị có hướng là số cung của đồ thị đi ra khỏi
nó (đi vào nó) và ký hiệu là deg  v và deg  v .
-) Định lý:
Giả sử G V , E  là đồ thị có hướng. Khi đó  deg  v   deg  v  E .
vV

vV

-) Đồ thị đầy đủ có n đỉnh, ký hiệu K n , là đơn đồ thị vô hướng mà giữa hai đỉnh bất kì của
nó luôn có cạnh nối.


Đồ thị đầy đủ K 6
-) Một clique trong đơn đồ thị vô hướng G  V , E  là tập các đỉnh V ' ( V ' là tập con của
tập các đỉnh V của G ) thoả mãn: với mỗi cặp đỉnh thuộc V ' luôn tồn tại một cạnh của
G nối chúng. Do vậy một đồ thị con được tạo ra từ V ' sẽ là một đồ thị đầy đủ. Kích thước
của một clique là số đỉnh của nó.

Nếu đồ thị đầy đủ K 5 là một đồ thị con thì tập đỉnh của nó là một clique kích thước 5
Bài 1: (Russia 2016)
Có 30 đội trong giải bóng rổ NBA và mỗi đội đấu 82 trận trong một mùa giải. Ban tổ chức
giải NBA muốn chia các đội thành bảng miền Tây và miền Đông (số đội trong mỗi giải
không nhất thiết phải bằng nhau) sao cho số trận đấu giữa các đội khác bảng bằng đúng một
nửa tổng số trận đấu. Hỏi Ban tổ chức có thể làm được như vậy không?
Giải:
Gọi số đội trong bảng miền Tây bằng x .
Tổng số trận đấu là 1230, do đó số trận đấu giữa các đội khác bảng bằng 615.

Từ đó ta phải có 82 x  615 bằng hai lần số trận đấu giữa các đội trong bảng miền Tây với
nhau, mâu thuẫn do 82 x  615 là một số lẻ.
Bài 2: (MMC 2015)
Trong một cuộc thi toán có một số thí sinh quen biết lẫn nhau. Chứng minh rằng tồn tại một
tập con M các thí sinh sao cho mỗi thí sinh trong M có nhiều nhất 3 người quen trong M
và mỗi thí sinh ngoài M có ít nhất 4 người quen trong M .
Giải:


Lấy tập con M gồm nhiều nhất có thể các thí sinh sao cho mỗi thí sinh trong M có nhiều
nhất 3 người quen trong M . Khi đó mọi thí sinh nằm ngoài M phải có ít nhất 4 người quen
trong M (nếu có thí sinh nào chỉ có nhiều nhất 3 người quen thì ta đưa thí sinh đó vào M ).
Đó chính là tập M cần tìm.
Bài 3: (Baltic Way 2015)
Có 100 thành viên trong một câu lạc bộ quý bà. Mỗi người đã uống trà (riêng) với đúng 56
người khác. Biết rằng có một nhóm A gồm 50 thành viên mà trong đó, hai người bất kỳ đã
uống trà với nhau. Chứng minh rằng có thể chia toàn bộ 100 thành viên câu lạc bộ này thành
hai nhóm, mỗi nhóm 50 thành viên sao cho trong mỗi nhóm, hai người bất kỳ đã uống trà
với nhau.
Giải:
Gọi mỗi quý bà là một đỉnh của đồ thị, mỗi cạnh nối hai đỉnh là hai quý bà đã uống trà với
nhau. Như vậy bậc của mỗi đỉnh bằng đúng 56.
Nếu chỉ xét riêng trong nhóm A thì bậc của mỗi đỉnh bằng 49, như vậy mỗi đỉnh này còn có
7 cạnh nối với các đỉnh ngoài nhóm A. Do đó có tổng cộng 350 cạnh như vậy.
Xét nhóm B gồm các thành viên còn lại của câu lạc bộ. Tổng số bậc của các đỉnh trong
nhóm B là 50  56  2800 .
Do có 350 cạnh nối các đỉnh giữa hai nhóm A, B nên nếu chỉ xét riêng trong nhóm B thì
tổng số bậc là 2450, tức là mỗi đỉnh trong nhóm B đều được nối với tất cả 49 đỉnh còn lại.
Đó chính là hai nhóm A, B cần tìm.
Bài 4: (Turkey 2014)

5 hãng hàng không mở các chuyến bay trong một đất nước có 36 thành phố. Giữa hai thành
phố bất kỳ có đúng một hãng hàng không mở một chuyến bay hai chiều. Nếu có một hãng
hàng không mở các chuyến bay nối các thành phố A với B và B với C thì ta gọi bộ ba
 A, B, C  là “cùng hãng”. Tìm giá trị lớn nhất của k sao cho trong mọi tình huống luôn có ít
nhất k bộ ba cùng hãng.
Giải:
5

Xét một đỉnh v bất kỳ, kề với xi cạnh của hãng hàng không Ci với 1  i  5 và

x

i

 35 .

i 1

5

Khi đó số các bộ ba cùng hãng nhận v là đỉnh ở giữa là

 xi 

7

  2   5  2   105 (theo bất đẳng

 
 

thức Jensen với đẳng thức xảy ra khi và chỉ khi x1  x2  x3  x4  x5  7 ). Do đó số các bộ
i 1

ba cùng hãng ít nhất là k  36  105  3780 .
 36 
Ngoài ra dễ thấy rằng   cạnh có thể chia thành 35 nhóm, mỗi nhóm 18 cạnh sử dụng đủ
2
36 đỉnh khác nhau. Mỗi hàng hàng không có thể quản lý 7 nhóm. Và như vậy mô hình này
thỏa mãn k  3780 .
Vậy k  3780 .


Bài 5: (IzhO 2016)
Có 60 thành phố trong xứ sở Đồ thị, trong đó hai thành phố bất kỳ đã được nối với nhau bởi
duy nhất một con đường một chiều. Chứng minh rằng ta có thể tô màu đỏ 4 thành phố và tô
màu xanh 4 thành phố sao cho mọi con đường nối các thành phố đỏ với các thành phố xanh
đều có chiều từ thành phố đỏ sang thành phố xanh.
Giải:
Giả sử phản chứng. Với mỗi đỉnh v của đồ thị, ký hiệu deg  v là bậc ra của nó.
Số các bộ

 X ,  A, B, C , D  

sao cho A, B, C , D có các cạnh bắt đầu từ X đến bằng

  deg  v 
 59 
 vV

 deg  v 

 


 , và theo bất đẳng thức Jensen thì không nhỏ hơn 60  60   60  2  , với
4
vV 



 4 
4


 r  r  r  1 r  2  r  3
.
 4 
1.2.3.4
 
Mặt khác mỗi bộ  A, B, C , D  xuất hiện nhiều nhất 3 lần, nên số các bộ  X ,  A, B, C , D  

 59 
 60 
 60 
như trên là không vượt quá 3   , mà 60  2   3   , mâu thuẫn.
 
4
4
 4 
Bài 6: (Iran 2015)
Một cái bánh hình tròn được chia thành 5 phần bất kỳ (có thể không bằng nhau) bởi các bán

kính. A và B muốn ăn cái bánh này. A ăn trước một phần bánh. Sau đó hai người sẽ tuần tự
ăn một phần bánh sao cho phần bánh tiếp theo chung bán kính với một phần bánh đã ăn
trước đó. Chứng minh rằng A luôn có cách để có thể ăn được tổng cộng ít nhất một nửa cái
bánh.
Giải:
Do B chỉ có thể ăn 2 phần bánh nên ta chỉ cần quan tâm đến 2 phần bánh mà có tổng diện
tích nhiều hơn một nửa cái bánh.
Gọi 5 phần bánh là A, B, C , D, E theo thứ tự kim đồng hồ. Xét đồ thị G nhận A, B, C , D, E
là các đỉnh, các cạnh nối giữa X , Y  V khi và chỉ khi X , Y có tổng diện tích nhiều hơn một
nửa cái bánh.
Khi đó ta chỉ có thể có nhiều nhất 3 đỉnh có cạnh nối trừ trường hợp tất cả các cạnh đều có
một đỉnh chung vì nếu không ta có thể chọn được 4 phần X , Y , Z ,W sao cho X , Y và Z ,W
đều có tổng nhiều hơn một nửa cái bánh, mâu thuẫn.
Nếu tất cả các cạnh đều có một đỉnh chung thì A chỉ cần chọn đỉnh chung đó đầu tiên và do
đó B không thể chọn được nhiều hơn một nửa cái bánh.
Nếu có nhiều nhất 3 đỉnh có cạnh nối và do như trên các cạnh này đều phải có đỉnh chung
thì A chỉ cần chọn một trong số các đỉnh chung này trong lượt đầu tiên, cụ thể ta có ba
trường hợp:


i) Nếu chỉ có hai cạnh XY và YZ thì A chọn Y đầu tiên.
ii) Nếu có một tam giác dạng ABC thì A chọn B đầu tiên, rồi B sẽ chọn A hoặc C , tiếp
theo A sẽ chọn đỉnh còn lại trong A, C ở lượt tiếp theo và thắng cuộc.
iii) Nếu có một tam giác dạng ACD thì A chọn A đầu tiên, tiếp theo A sẽ chọn C nếu B
chọn B và chọn D nếu B chọn E ở lượt tiếp theo và thắng cuộc.
Bài 7: (Hải Phòng TST 2015)
Cho đa giác lồi G có 2016 cạnh. X là tập chứa n  n  2  cạnh hay đường chéo của G sao cho
2 đoạn bất kì trong X đều có điểm chung. Tìm giá trị lớn nhất của n .
Giải:
Lập một đồ thị F như sau: mỗi đỉnh của G là một đỉnh của đồ thị. Hai đỉnh là kề nhau nếu

chúng là đầu mút của một đoạn thuộc X.
Kí hiệu mi là số đỉnh bậc i (với i  0;1;2;...;2016 ), ta có

m0  m1  m2  ...  m2015  2016

(1).

Xét đỉnh x có bậc i  2 , gọi x1 , x2 ,..., xi ( i  2 ) là các đỉnh kề với x sao cho các cạnh

( x, x j ) đều nằm trong miền góc tạo bởi hai cạnh ( x, x1 ) và ( x, xi ) 1  j  i.
Khi đó với mọi đỉnh y khác x thì đoạn ( x j , y ) không thể có điểm chung với cả hai cạnh

( x, x1 ) và ( x, xi ) , do đó ( x j , y ) không thuộc X nên không phải là cạnh của đồ thị F. Vậy
x j có bậc 1, 1  j  i .
Như vậy, mỗi đỉnh x bậc i  2 kề với ít nhất i  2 đỉnh bậc 1 và hiển nhiên mỗi đỉnh bậc 1
không thể kề với hai đỉnh khác nhau, vậy nên mi đỉnh bậc i  2 kề với ít nhất  i  2  mi đỉnh
bậc 1. Ta có bất đẳng thức sau:
m1  m3  2m4  3m5  ...  2013m2015

(2).

Mặt khác dễ có: 2n  m1  2m2  3m3  4m4  ...  2015m2015

(3).

Từ (1), (2), (3) suy ra:
2n  m1  2m2  3m3  4m4  ...  2015m2015

  m1  2m2  2m3  ...  2m2015    m3  2m4  3m5  ...  2013m2015 
  m1  2m2  2m3  ...  2m2015   m1  2  m1  m2  ...  m2015   2.2016

Vậy n  2016.
Bây giờ xét tập X gồm 2016 cạnh và đường chéo như sau:
 x1 , x2  ,  x1 , x3  ,  x1 , x4  ,...,  x1 , x2016  và  x2 , x2016  . Dễ thấy 2 đoạn bất kỳ thuộc X đều có
điểm chung.
Đáp số nmax  2016.
Bài 8: (Japan 2016)

3
 m  1 . Trong một đất nước có m
2
thành phố và n con đường hai chiều, mỗi con đường nối hai thành phố với nhau, hai thành
Cho các số nguyên dương m, n sao cho m  2, n 


phố có thể có nhiều hơn một con đường nối. Chứng minh rằng tồn tại một cách chia các
thành phố thành hai nhóm A và B sao cho tất cả các con đường nối một thành phố trong A
với một thành phố trong B sẽ được nâng cấp thành cao tốc sao cho:
i) Mỗi nhóm có ít nhất một thành phố.
ii) Mỗi thành phố chỉ có tối đa một đường cao tốc.
Giải:
Các trường hợp ban đầu là hoàn toàn rõ ràng.
Ta chứng minh bằng quy nạp theo số đỉnh của đồ thị kết hợp với giả sử phản chứng.
3
Giả sử ta có một đồ thị G  V , E  thỏa mãn E   V  1 .
2
Nếu có đỉnh chỉ có bậc không vượt quá 1 thì ta chỉ cần chọn A là đỉnh đó còn B là tập các
đỉnh còn lại.
Bây giờ ta xét trường hợp mỗi đỉnh đều có bậc nhỏ nhất là 2. Ta có một số nhận xét sau:
1) Một đỉnh bậc 2 không thể có cạnh kép tới một đỉnh khác.
Xét hai đỉnh a, b mà deg a  2 và có hai cạnh nối a, b . Xóa đỉnh a và các cạnh nối với a .

Đồ thị còn lại vẫn thỏa mãn điều kiện bài toán, do đó theo quy nạp ta có thể chia đồ thị còn
lại đó thành hai tập A’, B’ thỏa mãn. Sau đó ta bổ sung a vào tập chứa b để được cách phân
chia thỏa mãn.
2) Một đỉnh bậc 2 không thể nối với một đỉnh bậc 2 khác.
Xét hai đỉnh a, b được nối với nhau mà deg a  deg b  2 . Khi đó phải có đỉnh c được nối
với cả a và b (nếu không ta chỉ cần chọn A  a; b , B  V \ A ). Xóa hai đỉnh a, b và các
cạnh nối với chúng. Ta đã xóa 2 đỉnh và 3 cạnh nên đồ thị còn lại G  V , E  vẫn thỏa mãn

3
 V   1 , do đó theo quy nạp ta có thể chia đồ thị còn lại đó thành hai tập A’, B’ thỏa
2
mãn. Sau đó ta bổ sung a, b vào tập chứa c để được cách phân chia thỏa mãn.
3) Một đỉnh bậc 2 không thể nối với một đỉnh bậc 3.
Xét hai đỉnh a, b được nối với nhau mà deg a  2,deg b  3 . Theo 1), khi đó a còn phải
được nối với một đỉnh c khác b và b còn phải được nối với hai đỉnh x, y khác a (với x có
thể trùng y và có thể trùng c ). Bây giờ xóa a, b và các cạnh nối với chúng. Ta đã xóa 2
đỉnh và 4 cạnh nên vẫn có thể nối thêm một cạnh nữa vào để được đồ thị G  V , E  thỏa
E 

3
 V   1 .
2
Nếu x, y phân biệt, ta thêm 1 cạnh nối x, y để được đồ thị G  V , E  thỏa mãn
mãn E  

3
 V   1 , do đó theo quy nạp ta có thể chia đồ thị còn lại đó thành hai tập A’, B’ thỏa
2
mãn (nếu x  y thì không cần thêm cạnh).
Bây giờ bổ sung a vào tập đang chứa các đỉnh kề với a (trừ b ).

Nếu x, y cùng thuộc một trong hai tập đó thì bổ sung b vào cùng tập chứa x, y . Dễ thấy
cách bổ sung này thỏa mãn.
E 


Nếu x, y không ở trong cùng một tập thì bổ sung b vào cùng tập với a . Không mất tổng
quát giả sử x ở khác tập so với b . Khi đó cách phân chia này làm cho x có bậc (ngoài) là 2
khi nối với y và b . Nhưng x có một “cạnh ảo” với y nên cách phân chia này sẽ thỏa mãn
khi ta xóa “cạnh ảo” đó đi.
4) Hai đỉnh bậc 2 không thể nối tới cùng hai đỉnh.
Xét hai đỉnh a, b được nối với hai đỉnh c, d mà deg a  deg b  2 . Khi đó ta xóa a, b và các
cạnh nối với chúng. Ta đã xóa 2 đỉnh và 4 cạnh nên vẫn có thể nối thêm một cạnh nữa giữa
3
c, d vào để được đồ thị G  V , E  thỏa mãn E    V   1 , do đó theo quy nạp ta có
2
thể chia đồ thị còn lại đó thành hai tập A’, B’ thỏa mãn.
Nếu c, d ở trong cùng một tập thì bổ sung a, b vào cùng tập đó.
Nếu c, d không ở trong cùng một tập thì bổ sung a, b vào hai tập khác nhau, chẳng hạn a
cùng tập với c , b cùng tập với d . Khi đó cách phân chia này làm cho c và d đều có bậc
(ngoài) là 2 khi c nối với b và d còn d nối với a và c . Nhưng cạnh nối giữa c, d là
“cạnh ảo” nên cách phân chia này sẽ thỏa mãn khi ta xóa “cạnh ảo” đó đi.
5) Một đỉnh có bậc k  4 không thể nối với k  1 đỉnh bậc 2.
Xét đỉnh a có deg a  k  4 mà a nối với k  1 đỉnh bậc 2 là b1 ,..., bk 1 và a nối với đỉnh

c . Khi đó phải có đỉnh d được nối với 2 trong số các đỉnh b1 ; ...; bk 1; a (nếu không ta chỉ
cần chọn A  b1 ; ...; bk 1; a và B  V \ A ). Ngoài ra từ 4), d phải được nối với a và một
đỉnh khác, chẳng hạn là b1 .
Bây giờ xóa a, b1 và các cạnh nối với chúng. Ta đã xóa 2 đỉnh và k  1 cạnh nên vẫn có thể
nối thêm k  2 cạnh nữa giữa d và b2 ,..., bk 1 vào để được đồ thị G  V , E  thỏa mãn


3
 V   1 , do đó theo quy nạp ta có thể chia đồ thị còn lại đó thành hai tập A’, B’ thỏa
2
mãn. Bổ sung a, b1 vào cùng một tập với d , khi đó bậc (ngoài) của d không vượt quá 1.
E 

Nếu tất cả b2 ,..., bk 1 ở trong cùng tập đó thì bài toán được giải quyết.
Nếu một trong số b2 ,..., bk 1 ở trong tập khác, chẳng hạn là b2 thì ta đã tăng bậc (ngoài) của
nó lên nhiều nhất 1 (bằng cách nối với d ), nhưng đó là “cạnh ảo” nên bài toán cũng được
giải quyết.
3 V 1
Cuối cùng sử dụng 2), 3) và 5), ta có thể tạo ra G có ít nhất
cạnh, mâu thuẫn. Do đó
2
chứng minh quy nạp được khẳng định. Bài toán được chứng minh.
Bài 9: (IMO Shortlist 2016)
Một đất nước có n  3 hòn đảo. Ban đầu, một hãng phà mở một số chuyến phà (hai chiều)
nối một số cặp hòn đảo sao cho với mọi cách chia các hòn đảo thành hai nhóm khác rỗng thì
luôn có hai hòn đảo khác nhóm được nối với nhau.


Sau mỗi năm hãng phà sẽ bỏ một chuyến phà nối hai hòn đảo X , Y nào đó và với các hòn
đảo mà nối với đúng một trong X hoặc Y thì sẽ mở thêm chuyến phà mới nối hòn đảo này
với hòn đảo còn lại trong X hoặc Y .
Biết rằng tại mọi thời điểm, với mọi cách chia các hòn đảo thành hai nhóm khác rỗng, hãng
phà cũng sẽ đóng tất cả các chuyến phà nối hai hòn đảo khác nhóm sau một vài năm. Chứng
minh rằng sẽ có lúc có một hòn đảo được nối với tất cả các hòn đảo còn lại bằng các chuyến
phà.
Giải:
Ban đầu ta lấy hai hòn đảo bất kỳ được nối với nhau a, b và cho a vào tập A , b vào tập B .

Do điều kiện đề bài nên không mất tính tổng quát sẽ có hòn đảo c được nối với a . Cho c
vào B . Ta nói rằng hai tập hòn đảo tạo thành “mạng” nếu mỗi hòn đảo trong tập này đều
được nối với một hòn đảo trong tập kia.
Tiếp theo ta sẽ đưa dần các hòn đảo còn lại vào A  B . Giả sử rằng ta đang có hai tập A, B
tạo thành mạng với 3  A  B  n . Mạng này sẽ không còn tồn tại khi và chỉ khi hãng phà
đóng một chuyến phà nối

a A

với

b  B . Khi đó ta xét

A '  a; b



B '   A  B  \ a; b . Khi đó B ' khác rỗng. Xét hòn đảo bất kỳ c  A \ a , do điều kiện
giữa A, B nên ta có c được nối với b . Nếu c không được nối với a trước khi hãng phà
đóng chuyến phà nối a, b thì sau đó chuyến phà nối c, a sẽ được mở thêm. Khi đó c sẽ
được nối với cả a và b . Khẳng định này hoàn toàn tương tự đối với mọi hòn đảo trong
B \ b . Do đó A ', B ' cũng tạo thành mạng và A ' B '  A  B . Như vậy các hòn đảo này
luôn có thể chia thành hai tập A, B tạo thành mạng.
Vì A  B  n nên có hòn đảo vẫn chưa được đưa vào A  B . Theo đề bài sẽ có lúc chuyến
phà nối a  A  B với d nằm ngoài A  B bị đóng. Không mất tổng quát ta giả sử a  A .
Khi đó mọi hòn đảo trong B đều phải được nối với d . Do đó ta có thể đưa d vào trong A
để các tập A, B mới vẫn tạo thành mạng và số phần tử của A  B sẽ tăng thêm 1. Quá trình
tiếp tục như vậy và do mọi hòn đảo có thể đưa vào A, B nên ta dẫn đến việc xét trường hợp

A B  n.

Giả sử rằng chuyến phà nối a  A với b  B bị đóng sau một số năm nào đó. Ta đưa a, b
vào A ' còn tất cả các hòn đảo còn lại vào B ' . Khi đó A ', B ' tạo thành mạng. Điều này chỉ
không còn đúng nữa khi và chỉ khi chuyến phà nối (không mất tính tổng quát) a với c  B '
bị đóng. Do điều này chắc chắn xảy ra nên tại thời điểm đó, đảo b sẽ được nối với tất cả các
đảo còn lại.
Đó chính là điều phải chứng minh.
II – Đồ thị hai phe:
-) Một đơn đồ thị vô hướng G  V , E  được gọi là hai phe (hay lưỡng phân) nếu tập đỉnh
của nó có thể chia thành hai tập con X và Y rời nhau sao cho bất kì cạnh nào của đồ thị
cũng nối một đỉnh của X với một đỉnh thuộc Y . Khi đó người ta còn gọi một tập (chẳng


hạn X ) là tập các đỉnh trái và tập còn lại (chẳng hạn Y ) là tập các đỉnh phải của đồ thị hai
phe G .
Nếu X  Y thì G được gọi là đồ thị hai phe cân bằng.
-) Đồ thị hai phe đầy đủ K m,n là đồ thị hai phe với các tập đỉnh X  m, Y  n sao cho mọi
đỉnh của X đều kề với mọi đỉnh của Y . Như vậy K m,n có mn cạnh.

Đồ thị hai phe
-) Định lý Hall:
Cho đồ thị hai phe X , Y . Với mỗi tập con A của X , gọi G  A là tập các đỉnh thuộc Y kề
với một đỉnh nào đó thuộc A . Khi đó điều kiện cần và đủ để tồn tại một đơn ánh f : X  Y
sao cho x kề f  x  là G  A   A với mọi A khác rỗng của X .
Chứng minh:
Điều kiện cần là hiển nhiên: Nếu tồn tại đơn ánh f thì với mỗi A   x1 ; x2 ;...; xr  , ta
có G  A chứa các phần tử phân biệt  f  x1  ; f  x2  ;...; f  xn  , do đó G  A   r  A .
Ta chứng minh điều kiện đủ bằng quy nạp theo X .
Khi X  1 , khẳng định là hiển nhiên.
Giả sử định lý đã đúng với các tập X với X  n . Giả sử bây giờ X  n . Ta xét hai trường
hợp:

1) Giả sử với mọi A  X , A  X , ta có G  A   A . Chọn một phần tử x0 bất kỳ thuộc X ,
theo điều kiện G  x0    1 , do đó tồn tại y0 thuộc Y kề với x0 . Ta đặt f  x0   y0 . Bây
giờ xét X '  X \  x0  và Y '  Y \  y0  , A  X ' và G '  A  là tập các đỉnh thuộc Y ' kề với
A . Khi đó G '  A  G  A  1  A .
Vì X '  X nên theo giả thiết quy nạp, tồn tại đơn ánh f : X '  Y ' sao cho f  x  kề x với
mọi x thuộc X ' . Bổ sung thêm f  x0   y0 ta được đơn ánh f : X  Y thỏa mãn yêu cầu
định lý.
2) Trong trường hợp ngược lại, tồn tại A  X , A  X sao cho G  A   A . Khi đó,
do A  X nên tồn tại đơn ánh f : A  G  A  . Xét X '  X \ A, Y '  Y \ G  A  . Xét B thuộc
X ' và G  B  là tập các đỉnh thuộc Y ' kề với B .


Nếu G  B   B thì ta có G  A  B   G  A   G  B   A  B  A  B , mâu thuẫn với
điều kiện định lý.
Như vậy ta có G  B   B với mọi B thuộc X ' . Theo giả thiết quy nạp, tồn tại đơn
ánh g : X '  Y ' sao cho g  x  kề với x . Như vậy, ta có thể xây dựng được đơn
ánh h : X  Y sao cho h  x  kề với x , cụ thể h  x   f  x  nếu x thuộc A và h  x   g  x 
nếu x thuộc X \ A .
Vậy định lý được chứng minh.
Bài 10: (Tuymaada 2014)
Trên đảo Lena có một ngôi làng, bên trái đảo Lena có một hòn đảo chứa m ngôi làng, bên
phải đảo Lena có một hòn đảo khác chứa n ngôi làng với gcd  m  1, n  1  1 . Hai làng bất
kỳ bị ngăn cách bởi nước (kể cả làng Lena) đều có một chuyến phà hai chiều nối chúng,
chuyến phà này được đánh số hiệu là một số nguyên dương. Cư dân trên tất cả các ngôi làng
đều khẳng định rằng các chuyến phà ở làng của mình đều được đánh số hiệu là các số
nguyên dương khác nhau liên tiếp. Chứng minh rằng ít nhất một trong số các khẳng định đó
là sai.
Giải:
Xét đồ thị G  K m ,n   x , trong đó x là làng Lena, K m,n là đồ thị hai phe đầy đủ với phe
trái L  m , phe phải R  n , gcd  m  1, n  1  1 và x được nối với tất cả các đỉnh khác.

Các cạnh được đánh số hiệu là các số nguyên dương. Giả sử phản chứng rằng với mỗi đỉnh
trong đồ thị thì tất cả các cạnh kề với đỉnh đó đều được đánh số hiệu tạo thành các số nguyên
dương liên tiếp.
Với mỗi đỉnh V gọi S  v  là tập tất cả các số hiệu của các cạnh kề với v . Không mất tính
tổng quát ta giả sử S  x   1;2;; m  n , gọi d  gcd  m  1, n  1  1 và ký hiệu

D  v   l  S  v  : d | l .
Ta có

n 1
v  L

d

 m  1 n  1  1
m 1
D v  
v  R
  D v   D v   D v  D  x  2
d
vG
vL
vR
 d
 m  n 
  d  khi v  x


 m  n  m 1 n 1



 1)
(vì D  x   
d
d
 d 
Nhưng kết quả trên phải là một số chẵn (bằng hai lần số cạnh) nên mâu thuẫn.
Bài 11: (CGMO 2014)


Có n sinh viên, mỗi sinh viên quen biết (hai chiều) với đúng d sinh viên nữ và d sinh viên
nam. Tìm tất cả các cặp  n; d  nguyên dương có thể.
Giải:
Xét đồ thị G gồm các đỉnh lấy trong tập hợp các sinh viên nam B và tập hợp các sinh viên
nữ G với B  b, G  g , b  g  n .
Đếm các cạnh nối tập B với tập G ta có bd  gd nên suy ra b  g 

n
, hay n chẵn.
2

Ngoài ra rõ ràng n  2d .





Đặt B  b0 ; b1;...; bn  , G   g 0 ; g1;...; g n  và xét các cạnh bi gi ; bi g i 1;; bi g i  d 1 với
1
1

2 
2 


n
n
1  i   1 . Khi đó xét đồ thị tạo bởi B và đồ thị tạo bởi G , mỗi đồ thị có
đỉnh, mỗi
2
2
n
n
đỉnh có bậc d nên ta phải có d   1 và  d chẵn.
2
2
n
Như vậy tất cả các cặp thỏa mãn là n chẵn, n  2  d  1 và  d chẵn.
2
Bài 12: (CGMO 2013)
Có m nữ và n nam, trong đó với hai nam và hai nữ bất kỳ luôn có ít nhất một cặp nam nữ
không quen nhau (hai chiều). Chứng minh rằng số cặp nam nữ (không phân biệt thứ tự) quen
n  n  1
nhau không vượt quá m 
.
2
Giải:
Do ta không quan tâm đến việc quen nhau của các bạn cùng giới nên ta giả sử rằng các bạn
cùng giới là không quen nhau và xét đồ thị hai phe G tạo bởi các m đỉnh nữ và n đỉnh nam.
n  n  1
Ta cần chứng minh số cạnh của đồ thị thỏa mãn điều kiện đề bài là E  m 

(và
2
m  m  1
tương tự E  n 
).
2
Đặt A là tập các bạn nữ mà mỗi bạn trong đó quen với đúng một bạn nam và B là tập các
bạn nữ mà mỗi bạn trong đó quen với nhiều hơn một bạn nam, ký hiệu A  a, B  b . Rõ
ràng ta có a  b  m và E  a   degg .
gB

Bây giờ ta đếm số N các bộ  g ;b1 ; b2  với g  B là bạn nữ, b1 , b2 là hai bạn nam phân
biệt mà g quen cả b1 , b2 . Theo đề bài, với mỗi một trong

n  n  1
cặp b1 ; b2  , luôn chỉ có
2


nhiều nhất một bạn nữ g quen cả b1 , b2 nên ta có N 

n  n  1
. Ngoài ra ta cũng có
2

n  n  1
.
2
Mặt khác ta có
b


degg 
 degg  
deg g  deg g  1 b gB
  E  a  E  a  b 
gB
N 
 

 1 
2
2
b
b
2b
gB




(theo bất đẳng thức Jensen)
2

Nên  E  a   b  E  a   bn  n  1  0 , từ đó ta có

Ea

b  b 2  4bn  n  1
2




b  b 2  2bn  n  1   n  n  1 

2

b

n(n  1)
2

2
n  n  1
n  n  1
m
Cuối cùng ta có E  a  b 
.
2
2
n  n  1
Đẳng thức xảy ra khi chỉ khi b 
, tức là mỗi cặp bạn nam luôn có đúng một bạn nữ
2
quen cả hai, và ngoài ra a  m  b .
Bài 13: (Korea 2014)
Trên một hòn đảo có n tòa lâu đài, mỗi tòa lâu đài là của đất nước A hoặc B. Mỗi tòa lâu đài
có một lính gác (lính gác tất nhiên cùng đất nước với tòa lâu đài). Có một số con đường hai
chiều nối một số cặp tòa lâu đài (có thể nối hai tòa lâu đài khác đất nước), hai tòa lâu đài có
đường nối như vậy được gọi là “kề nhau”. Chứng minh rằng hai khẳng định sau là tương
đương:

1) Nếu một số lính của đất nước B di chuyển sang tấn công lâu đài kề của đất nước A thì
một số lính của đất nước A có thể di chuyển sang phòng thủ (ở lâu đài kề của đất nước A)
sao cho số lính phòng thủ của đất nước A ở tất cả các lâu đài như vậy đều không ít hơn số
lính tấn công của đất nước B.
2) Với mọi tập bất kỳ X các lâu đài của đất nước A, số lâu đài của đất nước A mà ở trong
X hoặc kề với ít nhất một lâu đài ở trong X đều không ít hơn số lâu đài của đất nước B mà
kề với ít nhất một lâu đài ở trong X .
Giải:
Với tập S các lâu đài, ký hiệu N A  S  là tập các lâu đài của đất nước A mà ở trong S hoặc
kề với ít nhất một lâu đài ở trong S , tương tự với ký hiệu N B  S  .
1) suy ra 2):
Với mọi tập bất kỳ X các lâu đài của đất nước A, khi đất nước B sang tấn công tất cả các
lâu đài trong X thì đất nước B có thể sử dụng tối đa là N B  X  lính còn đất nước A có thể


sử dụng tối đa là N A  X  lính để phòng thủ. Do tại mỗi lâu đài trong X số lính của đất
nước A đều không ít hơn số lính của đất nước B nên tổng cộng lại sẽ có N A  X   N B  X  .
2) suy ra 1):
Với các cuộc tấn công của đất nước B, xây dựng đồ thị hai phe  S , T  , trong đó S gồm các
đỉnh là tất cả các lâu đài của đất nước A còn T gồm các đỉnh là tất cả các lâu đài của đất
nước A mà bị đất nước B tấn công, trong đó mỗi lâu đài bị tấn công bao nhiêu lần thì xuất
hiện trong T bấy nhiêu lần. Đất nước A phòng thủ thành công khi và chỉ khi tất cả các đỉnh
trong T đều có cạnh kề.
Xét một cuộc tấn công bất kỳ của đất nước B tương ứng với tập con X '  T và X  S . Ta
có X '  N B  X  . Ngoài ra 2) có nghĩa là N A  X   N B  X  . Và hơn nữa trong đồ thị hai
phe ta có N  X '  N A  X  . Do đó ta có N  X '  X ' . Từ đó theo định lý Hall ta có đất
nước A sẽ phòng thủ thành công.
Bài 14: (Hải Phòng TST 2016)
Trong một giải đấu cờ vua, có n  5 đấu thủ tham gia (hai đấu thủ bất kỳ đấu với nhau tối đa


 n2 
một trận). Chứng minh rằng nếu số trận đấu đã diễn ra không nhỏ hơn    2 thì luôn tồn
4
tại 5 đấu thủ A, B, C, D, E mà A đã đấu với cả 4 đấu thủ còn lại, ngoài ra B đã đấu với C và
D đã đấu với E. Khẳng định của bài toán còn đúng không nếu số trận đấu đã diễn ra nhỏ hơn
 n2 
 4   2?
 
Giải:
Coi mỗi đấu thủ là một đỉnh của đồ thị. Hai đỉnh kề nhau nếu hai đấu thủ đã đấu với nhau.
 n2 
Nếu số trận đã đấu là không vượt quá    1 thì ta xét phản ví dụ sau:
4
n
 n  1
Chia n đỉnh thành 2 nhóm, một nhóm   đỉnh và một nhóm 
đỉnh. Nối mỗi đỉnh
2
 2 
thuộc nhóm 1 với tất cả các đỉnh thuộc nhóm 2, ngoài ra chọn 2 đỉnh tùy ý trong nhóm 1 và

 n2 
 n   n  1
nối chúng với nhau. Như vậy số cạnh là   

1

 4   1 và rõ ràng không tồn tại 5
 2   2 
 

đỉnh thỏa mãn yêu cầu bài toán.
 n2 
Ta c/m nếu số cạnh là không nhỏ hơn    2 thì tồn tại 5 đỉnh thỏa mãn yêu cầu (1).
4
 25 
Với n  5 : Số ván đã đấu là    2  8 . Nếu mỗi đỉnh có bậc lớn nhất là 3 thì số cạnh tối
4
5.3
 8 (vô lý). Do đó tồn tại ít nhất một đỉnh có bậc ít nhất là 4, giả sử là a . Gọi
đa là
2


b, c, d , e là 4 đỉnh kề với a . Trong đồ thị con 4 đỉnh này luôn tồn tại 4 cạnh, suy ra phải tồn
tại 2 cạnh không chung đỉnh, giả sử là bc, de . Bài toán được khẳng định.
k2 
Giả sử (1) đã đúng đến n  k  1  k  6  . Xét đồ thị k đỉnh với đúng    2 cạnh.
4
k 
Nếu tồn tại 1 đỉnh có bậc không vượt quá   thì bỏ đỉnh này đi và xét đồ thị con gồm k  1
2
đỉnh còn lại. Khi đó tổng số cạnh của đồ thị con k  1 đỉnh này ít nhất là

  k  1 2 
k2 
k 
 k    k  1 
 k   k  1
 4   2   2   2   2    2   1  2   2   2   2   4 
 

  
 
 



 
Như vậy khẳng định đúng theo giả thiết quy nạp.
k 
Nếu tất cả các đỉnh đều có bậc ít nhất là    1 .
2
Nếu k  2l  l  3 thì tổng số cạnh ít nhất là

2l  l  1
k2 
 l  l  1  l 2  2     2 .
2
4

Nếu k  2l  1  l  3 thì tổng số cạnh ít nhất là

 2l  1 l  1  l
2

k2 
 l  1  2     2 .
4

Dấu bằng phải xảy ra nên l  3, k  7 và tất cả các đỉnh đều có
bậc 4.


a3

a2
a1

 72 
Xét đồ thị 7 đỉnh a1 , a2 ,..., a7 với số cạnh là    2  14 và
4
mỗi đỉnh đều có bậc 4. Giả sử a1 kề a2 , a3 , a4 , a5 . Vì a2 bậc 4
nên a2 phải kề ít nhất một trong các đỉnh a3 , a4 , a5 . Giả sử a2
kề a3 .

a5

a4

Nếu a4 kề a5 thì bài toán được khẳng định.
Nếu a4 không kề a5 . Vì a4 bậc 4 nên a4 phải kề với ít nhất một
trong hai đỉnh a2 , a3 , giả sử là a2 . Khi đó:

a6

a7

Nếu a5 kề a3 thì bài toán được khẳng định.
Nếu a5 không kề a3 thì a5 phải kề a2 (hình vẽ).
Xét thêm a6 , a7 : có a1 , a2 bậc 4, không kề với a6 , a7 ; nên a6 , a7 kề nhau về kề với a3 , a4 , a5 .
Suy ra các cạnh a4 a1 , a4 a2 , a4 a6 , a4 a7 , a1a2 , a6 a7 thỏa mãn.
Vậy bài toán được chứng minh.

III – Định lý Turan:
-) Định lý Turan:


G là một đồ thị n đỉnh mà không chứa đồ thị con đầy đủ K k . Khi đó số cạnh của G không

k  2 n2
 .
k 1 2
Chứng minh:
Ta chứng minh bằng quy nạp theo n .
vượt quá

Rõ ràng khẳng định đúng với n  1, k .
Giả sử khẳng định đã đúng đến n  1 đỉnh, ta xét đồ thị G có n đỉnh, không chứa K k và có
số cạnh lớn nhất có thể.
Khi đó G phải chứa K k 1 , vì nếu không thì ta có thể thêm một cạnh vào đồ thị G và được
một đồ thị mới vẫn không chứa K k nhưng có số cạnh lớn hơn, mâu thuẫn.
Đặt A  K k 1 , B  G \ A .
Khi đó số cạnh của G mà có cả 2 đỉnh đều thuộc A là E A 

 k  2  k  1 .

2
Do B có n  k  1 đỉnh, không chứa K k nên theo giả thiết quy nạp ta có số cạnh của G mà
2

k  2  n  k  1

có cả 2 đỉnh đều thuộc B là EB 

.
k 1
2
Để không tạo ra K k thì một đỉnh thuộc B chỉ có thể nối với nhiều nhất k  2 đỉnh trong A .
Do đó ta có số cạnh của G mà nối các đỉnh thuộc A với các đỉnh thuộc B là
E AB   n  k  1 k  2  .
Từ đó số cạnh của G không vượt quá

 k  2  k  1  k  2   n  k  1

2

  n  k  1 k  2  

2
k 1
2
Đó chính là điều phải chứng minh.
-) Định lý Mantel: (trường hợp riêng của định lý Turan khi k  3 )

k  2 n2

k 1 2

 n2 
G là một đồ thị n đỉnh mà không chứa tam giác. Khi đó số cạnh lớn nhất của G là   .
4
Hơn nữa đẳng thức xảy ra khi và chỉ khi G là đồ thị hai phe đầy đủ K  n   n  .
 2 ,n  2 
   


Bài 15: (Iran 2016)
Trong mặt phẳng có 24 con robot. Mắt mỗi con robot có góc nhìn 700 . Hỏi 24 con robot này
có thể nhìn thấy tổng cộng nhiều nhất bao nhiêu con robot?
Giải:
Xét đồ thị có 24 đỉnh, mỗi đỉnh là một con robot. Ta gọi cặp đỉnh i, j là “tốt” nếu i nhìn
thấy j và j cũng nhìn thấy i . Khi đó ta nối hai đỉnh i, j bằng một cạnh (không phân biệt
chiều) và gọi là cạnh “tốt”.
Dễ thấy trong đồ thị này không tồn tại đồ thị con K 4 .


23  24
 a cạnh để tạo thành một đồ
2
thị đầy đủ. Ta gọi các cạnh còn thiếu này là cạnh “xấu”.
Khi đó mỗi cạnh “tốt” tương ứng với 2 lần nhìn thấy, mỗi cạnh “xấu” tương ứng với nhiều
nhất 1 lần nhìn thấy. Do đó tổng số lần nhìn thấy nhiều nhất là 276  a  2a  276  a .
3
Ngoài ra theo định lý Turan ta có a  24  24  2   192 (vì trong đồ thị không có đồ thị
2
con K 4 ).
Giả sử đồ thị này có a cạnh “tốt”, khi đó ta còn thiếu

Do đó tổng số lần nhìn thấy nhiều nhất là 276  192  468 .
Bài 16: (Tournament of the Towns 2016)
a) Có 2n  1, n  2 cục pin. Ta không biết cục pin nào tốt, cục pin nào xấu, chỉ biết rằng số
cục pin tốt nhiều hơn số cục pin xấu là 1. Một cái đèn sử dụng hai cục pin và chỉ sáng nếu cả
hai cục pin đều tốt. Cần thử lắp pin để kiểm tra ít nhất bao nhiêu lần để chắc chắn là trong
mọi trường hợp thì đèn sáng?
b) Câu hỏi tương tự nếu có 2n, n  2 cục pin và số cục pin tốt bằng số cục pin xấu.

Giải:
a) Xét đồ thị hai phe có các đỉnh là các cục pin, n  1 đỉnh tốt và n đỉnh xấu, nối hai cục pin
bởi một cạnh nếu chúng đã được kiểm tra. Ta chỉ ra đáp số là n  2 .
Giả sử phản chứng với số lần kiểm tra là k  n  1 . Ta chỉ ra rằng tồn tại đồ thị 2n  1 đỉnh
với k  n  1 cạnh mà n  1 đỉnh tốt chưa có một cạnh nào.
Áp dụng định lý Turan cho đồ thị bù ta cần chứng minh

 n  1  2   2n  1
2
 n  1  1

2

 n2

 n
    2n 2  1  4n 2  2n  4n3  3n  1
 2
Khẳng định cuối này đúng.
Ta chỉ ra cách kiểm tra với k  n  2 :
Gọi các cục pin là B0 , B1 ,, B2 n rồi kiểm tra các cục pin  Bi ; Bni  với 1  i  n đến  B0 ; B1 
và  B0 ; Bn 1  . Rõ ràng rằng nếu n lần kiểm tra đầu tiên, đèn đều không sáng thì B0 tốt còn
các cục pin Bi , Bn i có một tốt, một xấu. Từ đó, một trong hai lần kiểm tra  B0 ; B1  hoặc

 B0 ; Bn1  sẽ bật sáng đèn.
b) Ta chỉ ra đáp số là n  3 .
2

n  2  2n 


 2n 2  2n  2 , khẳng định này
Áp dụng định lý Turan một cách tương tự ta có
n 1
2
cũng đúng.
Ta chỉ ra cách kiểm tra với k  n  3 :
Gọi các cục pin là B1 ,, B2 n rồi kiểm tra các cục pin B2 k 1 , B2 k với 4  k  n đến

 B1; B2  ,  B2 ; B3  ,  B1; B3  ,  B4 ; B5  ,  B5 ; B6  ,  B6 ; B4  . Dễ thấy cách kiểm tra này thỏa mãn.


Bài 17: (VMO 2017)
Cho số nguyên n  1 . Bảng vuông ABCD kích thước n  n gồm n 2 ô vuông đơn vị, mỗi ô
vuông đơn vị được tô bởi một trong ba màu đen, trắng hoặc xám. Một cách tô màu được gọi
là đối xứng nếu mỗi ô có tâm trên đường chéo AC được tô màu xám và mỗi cặp ô đối xứng
qua AC được tô cùng màu đen hoặc cùng màu trắng. Người ta điền vào mỗi ô xám số 0,
mỗi ô trắng một số nguyên dương và mỗi ô đen một số nguyên âm. Một cách điền số như
vậy được gọi là k  cân đối (với k là số nguyên dương) nếu thỏa mãn các điều kiện sau:
(i) Mỗi cặp ô đối xứng qua AC được điền cùng một số nguyên thuộc đoạn  k ; k  .
(ii) Nếu một hàng và một cột giao nhau tại ô đen thì tập các số nguyên dương được điền trên
hàng đó và tập số nguyên dương được điền trên cột đó không giao nhau; nếu một hàng và
một cột giao nhau tại ô trắng thì tập các số nguyên âm được điền trên hàng đó và tập các số
nguyên âm được điền trên cột đó không giao nhau.
a) Với n  5 , tìm giá trị nhỏ nhất của k để tồn tại cách điền hình số k  cân đối cho cách tô
màu như hình bên dưới

b) Với n  2017 , tìm giá trị nhỏ nhất của k để với mọi cách tô màu đối xứng, luôn tồn tại
cách điền k  cân đối.
Giải:
a) Gọi Ai là tập hợp các số nguyên dương trên hàng i với  i  1,5 .

Theo hình của đề bài ta cần có A1  A3  , A1  A4  , A3  A5   và ngoài ra

A1  A5  , A3  A4  , A4  A5   .
Với k  1 không thể điền được.
Với k  2 ta cần có A1  1 vì nếu A1  2 thì không còn số để chọn cho A3 . Giả sử A1  1
thì A3  A4  2 , A5  1 , mâu thuẫn với A4  A5   .
Ta chứng minh k  3 thỏa mãn với cách điền như sau
0 1 -1 -2 1
1 0 2 2 3
-1 2 0 2 -3
-2 2 2 0 3
1 3 -3 3 0
b) Điều kiện cần:
Xét cách tô màu xen kẽ đen trắng như bàn cờ (hình vẽ), trong đó vị trí  i; j  được tô đen nếu

i  j chẵn, ngược lại tô trắng.


Xét hai ô trắng bất kỳ  a; b  ,  c; d  với 1  a, b, c, d  2017 ở bảng ô vuông trên.
Nếu a  c và b  d cùng chẵn thì a  d và b  c cùng lẻ, do đó một trong hai ô  a; d  hoặc

 b; c  có màu đen. Suy ra hai ô trắng phải được điền số khác nhau.
Nếu a  c và b  d cùng lẻ thì xét ô  d ; c  điền cùng số với ô  c; d 

và cũng tương tự như

trên ta suy ra hai ô phải được điền số khác nhau.
Suy ra tất cả các ô trắng ở nửa trên bên phải của bảng phải được điền các số khác nhau. Do
2007 2  1
đó ta phải có k  2  4  6  ...  2016 

.
4
Điều kiện đủ:
2007 2  1
Ta sẽ chứng minh rằng k 
thỏa mãn bài toán bằng quy nạp rằng với bảng n  n
4

 n2 
thì đáp số là kn    .
4
Khẳng định là rõ ràng với n  1, n  2, n  3 .
Giả sử khẳng định đã đúng với mọi số nguyên dương nhỏ hơn n  4 .
Đánh số các hàng và cột từ 1 đến n . Ta chứng minh rằng với mọi vị trí các ô đen thì luôn
tồn tại cách điền các số nguyên dương không vượt quá kn vào các ô trắng còn lại trong bảng
(các số âm tương tự).
Xét đồ thị G có n đỉnh, đỉnh thứ i ứng với hàng i với i từ 1 đến n , có cạnh nối từ đỉnh i
đến đỉnh j khi và chỉ khi các ô  i; j  ,  j; i  màu trắng.
Áp dụng định lý Mantel – Turan ta có:

 n2 
Nếu G không chứa tam giác thì G có không quá   cạnh, hay là trong bảng có không
4
 n2 
 n2 
quá   ô trắng, do đó có thể sử dụng   số nguyên dương phân biệt để điền vào các ô
4
4
trắng.
Nếu G có chứa tam giác, chẳng hạn các đỉnh a, b, c được nối với nhau. Do đó các ô


 a; b  ,  b; c  ,  c; a  ,  b; a  ,  c; b  ,  a; c 

đều có màu trắng. Khi đó các số điền vào các ô trắng

đó không nhất thiết phân biệt và tập hợp các ô trắng còn lại (nếu có) trên các hàng a, b, c
cũng không nhất thiết phải rời nhau. Rõ ràng trên mỗi hàng còn lại không quá n  3 ô như


thế. Do đó ta có thể sử dụng một số để điền vào các ô trắng ở trên và không quá n  3 số để
điền vào các ô trắng còn lại trong hàng.
Nếu không tính các hàng a, b, c thì ta còn lại n  3 hàng, theo quy nạp ta cần không quá

  n  3 2 

 số nguyên dương cho các hàng đó.
 4 
  n  3 2    n  12   n 2 
Như vậy tổng cộng ta cần không quá 1  n  3  

    số nguyên
 4   4   4 
dương.
 n2 
Như vậy trong mọi trường hợp ta cần không quá   số nguyên dương, điều phải chứng
4
minh.
Bài 18: (China TST 2017)
2017 kỹ sư tham dự một hội thảo. Hai người bất kỳ nói chuyện với nhau bằng tiếng Anh
hoặc tiếng Trung. Không có hai người nào nói chuyện với nhau nhiều hơn một lần. Biết rằng

trong 4 kỹ sư bất kỳ luôn có một số chẵn cuộc nói chuyện (hai người), ngoài ra trong các
cuộc nói chuyện này luôn có:
i) Ít nhất một cuộc nói chuyện bằng tiếng Trung
ii) Hoặc không có cuộc nói chuyện bằng tiếng Anh nào hoặc số cuộc nói chuyện bằng tiếng
Anh không nhiều hơn số cuộc nói chuyện bằng tiếng Trung
Chứng minh rằng luôn tồn tại 673 kỹ sư mà hai người bất kỳ trong đó nói chuyện với nhau
bằng tiếng Anh.
Giải:
Bài toán đồ thị:
Gọi G là một đồ thị có 2017 đỉnh, hai đỉnh bất kỳ trong đó nếu được nối với nhau bằng một
cạnh thì cạnh đó sẽ được tô màu xanh hoặc đỏ. Khi đó trong 4 đỉnh bất kỳ trong G sẽ luôn
có một số chẵn cạnh, ngoài ra trong các cạnh này luôn có:
i) Ít nhất một cạnh màu đỏ
ii) Hoặc không có cạnh nào màu xanh hoặc số cạnh màu xanh không nhiều hơn số cạnh màu
đỏ
Chứng minh rằng luôn tồn tại 673 đỉnh mà hai đỉnh bất kỳ trong đó được nối với nhau bằng
một cạnh đỏ.
Giải:
Theo định lý Turan, số các cạnh đỏ trong G ít nhất là bằng
 2017   1  2017 2 2017 2 2017 2017.2014



 2   1   
2
6
2
6

  3

Do đó tổng số bậc đỏ trong G ít nhất là bằng

2017.2014
.
3


Từ đó theo nguyên lý Dirichlet ta có luôn tồn tại một đỉnh v mà bậc đỏ ít nhất là bằng
 1 2017  2014   2014 
 2017 
   3   672 .
3
Gọi v1 ,, v672 là các đỉnh được nối với v bằng một cạnh đỏ. Xét các chỉ số i  j  k  672
sao cho giữa các đỉnh {vi ; v j ; vk } có ít nhất một cạnh đỏ. Do các điều kiện i) và ii) đối với các
đỉnh c; vi ; v j ; vk  ta có một cạnh đỏ hoặc ba cạnh đỏ giữa các đỉnh {vi ; v j ; vk } .
Cũng theo i) ta có nhiều nhất ba thành phần liên thông trong đồ thị con H  v1;; v672 
theo nghĩa là bỏ qua các cạnh xanh.
Ngoài ra mối quan hệ “nối với nhau bằng một cạnh đỏ” cũng sẽ được giữ nguyên với ba
đỉnh bất kỳ trong đồ thị con H . Khi đó H sẽ là hợp của ba chu trình đỏ không giao nhau
A, B, C (mỗi chu trình này có thể rỗng).
Nếu chỉ có chẳng hạn A không rỗng thì v  A chính là chu trình đỏ với 673 đỉnh cần tìm.
Nếu có ít nhất hai trong ba tập A, B, C không rỗng. Không mất tính tổng quát ta giả sử

A  B  C thì A  3, B  1 .
Xét 4 đỉnh a, b, c, d , trong đó a, b, c  A và d  B . Do ad , bd , cd không có màu đỏ nên theo
ii) chúng phải có màu xanh. Do đó giữa các cạnh nối các đỉnh trong v; a; b; d  , hai cạnh
màu xanh và bốn cạnh màu đỏ, mâu thuẫn với ii).
IV – Chu trình, đường đi:
-) Đường đi độ dài n từ đỉnh u đến đỉnh v trên đồ thị vô hướng G V , E  là dãy


x0 , x1 ,..., xn1 , xn , trong đó n là số nguyên dương, x0  u , xn  v,  xi ; xi 1   E i  0, n  1 .
Đường đi như trên còn có thể biểu diễn thành dãy các cạnh  x0 ; x1  ,  x1; x2  ,...,  xn1 ; xn  .
Đỉnh u là đỉnh đầu, đỉnh v là đỉnh cuối của đường đi. Đường đi có đỉnh đầu trùng với đỉnh
cuối ( u  v ) được gọi là chu trình. Đường đi hay chu trình được gọi là đơn nếu như không
có cạnh nào lặp lại.
-) Khái niệm đường đi và chu trình trên đồ thị có hướng được định nghĩa hoàn toàn tương tự,
chỉ có điều khác biệt duy nhất là ta phải chú ý tới các cung của đồ thị.
Bài 19: (Russia 2014)
Một đất nước có n thành phố, mỗi cặp thành phố đều có chuyến tàu cao tốc hai chiều nối
chúng. Các chuyến tàu đi về giữa cùng hai thành phố thì có giá vé bằng nhau còn các chuyến
tàu giữa các thành phố khác nhau thì có giá vé khác nhau. Chứng minh rằng du khách có thể
chọn được một hành trình trên n  1 chuyến tàu sao cho giá vé các chuyến tàu là giảm dần
(du khách có thể đi qua một thành phố nào đó nhiều lần).
Giải:
Thuật toán Greedy:
Bắt đầu từ một đỉnh v1 , du khách sẽ đi sang đỉnh v2 sao cho giá của chuyến tàu v1  v2 là
cao nhất trong số các đỉnh kề của v1 , tiếp tục như vậy đến khi không thể, du khách sẽ kết


thúc hành trình tại một đỉnh vk nào đó. Áp dụng thuật toán trên tại tất cả các đỉnh, ta sẽ có n
hành trình khác nhau.
Bổ đề:
Với hai đỉnh U ,V bất kỳ, luôn có ít nhất một hành trình như trên chứa chuyến tàu đi từ
U V .
Chứng minh:
Chỉ cần đảo ngược lại thuật toán Greedy: đặt V  A0 ,U  A1 , ta chọn A2 sao cho giá của
chuyến tàu A2  A1 là nhỏ nhất và cao hơn giá của chuyến tàu U  V , tiếp tục như vậy đến
khi không thể, quá trình sẽ kết thúc tại một đỉnh At nào đó, và khi đó ta có hành trình

At  At 1    A1  A0 thỏa mãn.

Áp dụng:

 n
2 
2
Theo nguyên lý Dirichlet, sẽ luôn tồn tại một hành trình có độ dài ít nhất là    n  1
n
thỏa mãn đề bài.
Bài 20: (Conor Sur 2013)
Nocycleland là một đất nước có 500 thành phố và 2013 con đường hai chiều nối các cặp
thành phố. Thành phố A gọi là “kề” với thành phố B nếu có con đường trực tiếp nối chúng,
và gọi là “gần kề” với B nếu có thành phố C mà A kề C và C kề B. Biết rằng trong
Nocycleland, mỗi cặp thành phố chỉ được nối trực tiếp với nhau bởi không quá một con
đường, ngoài ra không có bốn thành phố A, B, C, D mà A kề B, B kề C, C kề D và D kề A.
Chứng minh rằng tồn tại một thành phố gần kề với ít nhất 57 thành phố khác.
Giải:
Với 500  V và 2013  E . Với mỗi đỉnh v gọi d  v  là số các đỉnh kề với v và d 2  v  là số
các đỉnh gần kề với v . Do không có chu trình bậc 4 nên nếu A gần kề với B thì chỉ tồn tại
duy nhất C mà A kề C và C kề B.
Do đó d 2  v     d  w   1 . Từ đó  d 2  v      d  w   1   d  w   d  w   1 .
w kÒ v

Do

 d  w  2 E và  d  w

v

2




 2E 

V
Áp dụng ta có điều phải chứng minh.

v w kÒ v

2

nên

 2E 
 d v  V
2

w

2

 2E .

Bài 21: (International Olympiad of Metropolises 2016)
Một đất nước có n thành phố, một số cặp thành phố được nối với nhau bởi các chuyến bay
một chiều của một trong hai hãng hàng không A hoặc B (mỗi cặp thành phố có thể có nhiều
hơn một chuyến bay cùng chiều). Một dãy gồm các ký tự A, B được gọi là “thực hiện được”
nếu tồn tại một chuỗi hành trình liên tiếp giữa các thành phố tuần tự bằng các hãng hàng
không A hoặc B tương ứng với dãy đó, chẳng hạn dãy AAB là “thực hiện được” nếu tồn tại
4 thành phố không nhất thiết phân biệt a1 , a2 , a3 , a4 mà đi từ a1 đến a2 bằng hãng hàng



không A, từ a2 đến a3 bằng hãng hàng không A và từ a3 đến a4 bằng hãng hàng không B.
Biết rằng tất cả các dãy ký tự có độ dài 2n đều “thực hiện được”, chứng minh rằng mọi dãy
ký tự độ dài hữu hạn đều “thực hiện được”.
Giải:
P là tập hợp gồm các tập con của tập tất cả các thành phố. Khi đó P có 2n phần tử.
Với S , T là hai tập con của tập tất cả các thành phố, ta nối S , T bởi một cạnh và đặt tên là A
nếu tồn tại một thành phố a trong S sao cho tập T chính là tất cả các thành phố b mà bắt
đầu từ a , sử dụng chuyến bay của hãng hàng không A thì đến được b . Tương tự với các
cạnh đặt tên là B . Như vậy tập P trở thành đồ thị có hướng với các đỉnh là các tập con của
tập tất cả các thành phố.
Bây giờ mỗi dãy ký tự tương ứng với một đường đi trong P bắt đầu từ tập tất cả các thành
phố. Dãy ký tự là “thực hiện được” nếu đường đi này không dẫn đến tập rỗng.
Có 2n phần tử trong P , do đó bắt đầu từ tập tất cả các thành phố, để dẫn đến tập rỗng thì
đường đi ngắn nhất có độ dài lớn nhất là 2n  1, tức là những dãy “không thực hiện được”
đều có thể đưa về dãy “không thực hiện được” mà có độ dài không quá 2n  1.
Tuy nhiên rõ ràng một dãy độ dài không quá 2n  1 luôn có thể bổ sung thêm một số ký tự
nữa (vào cuối dãy chẳng hạn) để trở thành dãy độ dài 2n và “thực hiện được”.
Như vậy mọi dãy ký tự độ dài hữu hạn đều “thực hiện được”.
Đó chính là điều phải chứng minh.
Bài 22: (Hong Kong TST 2017)
Một ủy ban có n thành viên. Mỗi cặp thành viên là bạn hoặc kẻ thù. Mỗi thành viên có đúng
3 kẻ thù. Biết rằng với mỗi thành viên, kẻ thù của bạn anh ta thì cũng là kẻ thù của anh ta.
Tìm tất cả các giá trị có thể của n .
Giải:
Chú ý rằng trong ba người bất kỳ không thể có đúng một cặp kẻ thù.
Ngoài ra, không thể có một chu trình 4 người là bạn bởi vì họ phải có ít nhất một kẻ thù
chung nên từ đó kẻ thù chung đó sẽ có ít nhất 4 kẻ thù (là 4 người đó). Do đó mỗi nhóm 4
người luôn phải có ít nhất hai cặp kẻ thù.

Ngoài ra mỗi nhóm 4 người cũng không thể chỉ có đúng hai cặp kẻ thù, ví dụ nếu ta xét 4
người A, B, C , D : nếu AB, AC là hai cặp kẻ thù hoặc nếu AB, CD là hai cặp kẻ thù thì nhóm
ABD đều sẽ mâu thuẫn.
Nếu trong nhóm A, B, C , D có 3 cặp kẻ thù thì các cặp đó phải có dạng AB, AC , AD hoặc
AB, BC , CA và nếu có 4 cặp kẻ thù thì các cặp đó phải có dạng AC , AD, BC , BD , còn các
trường hợp có 5 hoặc 6 cặp kẻ thù đều thỏa mãn.
Bây giờ ta khẳng định rằng n  4,5, 6 .
Rõ ràng n  4 không thỏa mãn vì mỗi người phải có đúng 3 kẻ thù.
Với n  4 là khi mọi cặp hai người đều là kẻ thù.
Với n  5 là khi AB, AC , AD, AE , BC , BD, BE là các cặp kẻ thù.
Với n  6 là khi đồ thị K 3,3 là các cặp kẻ thù.


Với n  6 , chú ý rằng không có 4 người nào có thể tạo thành một chu trình đầy đủ kẻ thù
hoặc thậm chí là 5 cặp kẻ thù bởi vì hai người trong đó sẽ có 3 kẻ thù bao gồm cả người kia,
từ đó sẽ có những người còn lại là bạn chung, mâu thuẫn.
Xét một người A và ba kẻ thù D, E , F thì DEF phải tạo thành một chu trình bạn. Nhưng
khi đó mọi kẻ thù khác của D, E , F phải là chung cho cả ba người, do đó ta có các tam giác
bạn ABC , DEF và các kẻ thù K 3,3 giữa họ. Khi đó mọi người khác phải là bạn của A , do đó
phải là kẻ thù của D , điều này khiến D có nhiều hơn 3 kẻ thù, mâu thuẫn.
Bài 23: (IMO Shortlist 2013)
Trong một đất nước, một số cặp thành phố được nối với nhau bằng các chuyến bay hai chiều
sao cho hai thành phố bất kỳ luôn được nối với nhau (có thể là thông qua một chuỗi các
chuyến bay trung gian). Khoảng cách giữa hai thành phố được định nghĩa là số chuyến bay ít
nhất để nối chúng. Biết rằng với mỗi thành phố có nhiều nhất 100 thành phố có khoảng cách
đến nó bằng đúng 3. Chứng minh rằng không có thành phố nào có nhiều hơn 2550 thành phố
có khoảng cách đến nó bằng đúng 4.
Giải:
Xét bài toán tổng quát với việc thay số 100 thành số l bất kỳ.
Xét một đồ thị G với các đỉnh là các thành phố, mỗi cạnh biểu thị cho đường bay nối hai

thành phố.
Xét một đỉnh bất kỳ v  V  G  , gọi d 4  v  là số các đỉnh có khoảng cách bằng đúng 4 đến v .
Gọi

v ; v ;; v 
1

2

p

là tập các đỉnh có khoảng cách bằng đúng 1 đến v . Với mỗi

i  1;2;...; p , gọi Vi là tập các đỉnh w không nằm trong V j j  i với khoảng cách 4 từ v
mà tồn tại một con đường độ dài 3 không chứa v nối vi với w , đặt Vi  ni .
Nếu mọi ni đều bằng 0 thì d 4  v   0 . Do đó không mất tổng quát ta giả sử n1 , n2 ,, nq khác
0 (nếu không thì ta đánh số lại).
Bây giờ chú ý rằng tất cả các đỉnh trong Vi đều có khoảng cách 3 đến vi , và với mỗi

j  1;2;...; q \ i luôn có thêm ít nhất một đỉnh có khoảng cách 3 đến vi .
Tiếp theo lại do không đỉnh nào có nhiều hơn l đỉnh có khoảng cách 3 đến nó nên với mọi
i  1;2;...; q ta có ni  q  1  l .
q

q

2

 l 1
Và ta có d 4  v    ni    l  1  q    l  1  q  q  

 .
 2 
i 1
i 1

 l  1  2 
Do đó số đỉnh có khoảng cách 4 đến một đỉnh cho trước không vượt quá 
 .
 2  
Áp dụng với l  100 ta có điều phải chứng minh.
V – Liên thông:
-) Một đồ thị vô hướng được gọi là liên thông nếu có đường đi giữa mọi cặp đỉnh phân biệt
của đồ thị.


-) Cho đồ thị G V , E  và v  V . V ' là tập hợp các đỉnh của V liên thông với v , E ' là tập
hợp các cạnh nối 2 đỉnh của V ' . Khi đó đồ thị G ' V ', E ' gọi là thành phần liên thông của
G chứa v . Đương nhiên nếu v và u liên thông trong G thì thành phần liên thông của G
chứa v cũng là thành phần liên thông chứa u .
-) Đồ thị G V , E  là liên thông khi và chỉ khi G có duy nhất một thành phần liên thông.

-) Đồ thị có hướng G V , E  được gọi là liên thông mạnh nếu có đường đi từ a đến b và từ

b đến a với  a, b  V .
-) Đồ thị có hướng G V , E  được gọi là liên thông yếu nếu đồ thị vô hướng tương ứng của
nó là liên thông.
-) Đồ thị G V , E  được gọi là k  liên thông ( k  2 ) nếu có ít nhất k đỉnh và nếu xóa đi k  1
đỉnh bất kỳ cùng với các cạnh của nó thì đồ thị còn lại vẫn liên thông.

Đồ thị 2  liên thông


Đồ thị 4  liên thông

Bài 24: (EGMO 2016)
Với số nguyên dương m cho trước, xét bảng 4m  4m ô vuông đơn vị. Hai ô vuông phân
biệt được gọi là “liên quan” với nhau nếu chúng cùng hàng hoặc cùng cột. Tô màu xanh một
số ô vuông sao cho mỗi ô vuông trong bảng đều “liên quan” với ít nhất hai ô vuông xanh
(không kể chính ô vuông đó nếu nó màu xanh). Hỏi có ít nhất bao nhiêu ô vuông màu xanh?
Giải:
Ta sẽ chứng minh rằng đáp số là 6m .
Nếu có dòng (hoặc cột) nào đó không có ô vuông xanh thì rõ ràng ta cần có ít nhất 8m ô
vuông xanh (mỗi cột (hoặc dòng) cần có ít nhất 2 ô vuông xanh).
Bây giờ nếu tất cả các dòng và cột đều có ô vuông xanh. Xét đồ thị hai phe với mỗi đỉnh là
dòng hoặc cột, còn cạnh là ô vuông xanh (ví dụ ô vuông ở dòng i cột j màu xanh thì ta nối
đỉnh di với đỉnh c j ).
Do mỗi dòng và cột đều có ô vuông xanh nên mỗi thành phần liên thông đều có ít nhất 1
cạnh.
Ngoài ra do điều kiện đề bài nên mỗi cạnh đều kề với ít nhất hai cạnh khác, do đó mỗi thành
phần liên thông đều có ít nhất 3 cạnh.
Do tam giác không phải đồ thị hai phe nên mỗi thành phần liên thông phải có ít nhất 4 đỉnh.
V 1
3
V  V .
Trong mỗi thành phần liên thông có V đỉnh, E cạnh thì E  V  1 
V
4


Tài liệu bạn tìm kiếm đã sẵn sàng tải về

Tải bản đầy đủ ngay
×